What’s the correct answer for this question?

Whats The Correct Answer For This Question?

Answers

Answer 1

Answer:

B.

Step-by-step explanation:

Volume of the model of moon = 4/3(πr³)

= 4/3(π)(1)³

= 4.2 feet³

Volume of cylinder = πr²h

= (3.14)(0.5)²(0.5)

= 0.39 feet³

Cylindrical clay boxes to be used = 4.2/0.39

=10.7 ≈ 11


Related Questions

In Evan's senior class of 240 students, 85% are planning to attend college after graduation. What is the
probability that a senior is chosen at random not planning to attend college after graduation?

Answers

Answer:

For this case we want to find the probability that a senior is chosen at random not planning to attend college after graduation so then we can use the complement rule given by:

[tex] P(A') = 1-P(A)[/tex]

Where A is the vent of interest and replacing we got:

[tex] P(A') = 1-0.85= 0.15[/tex]

So then the probability that a senior is chosen at random not planning to attend college after graduation is 0.15 or 15%

Step-by-step explanation:

For this case we know that the sample size if n =240 and we also know that the probability that in the Evan's class the any student are planeed to attend collge after graduation is:

[tex] P(A) =0.85[/tex]

For this case we want to find the probability that a senior is chosen at random not planning to attend college after graduation so then we can use the complement rule given by:

[tex] P(A') = 1-P(A)[/tex]

Where A is the vent of interest and replacing we got:

[tex] P(A') = 1-0.85= 0.15[/tex]

So then the probability that a senior is chosen at random not planning to attend college after graduation is 0.15 or 15%

In 2013, the average Girl Scout in New York City sold 96 boxes of cookies. The leader of Troop 5078 in New York City wants to know if the scouts in her troop sold more cookies than the average in New York City. She randomly samples 50 girls in Troop 5078 and records the number of boxes of cookies sold for each girl in the sample. The troop leader finds that her Girl Scouts each sold an average of 101.1 boxes of cookies with a standard deviation of 29.3. She analyzed her data using a t-test and obtained a p-value of 0.11. What conclusion can she draw from her data?

Answers

Answer:

We want to test if the scouts in her troop sold more cookies than the average in New York City (50), the system of hypothesis would be:  

Null hypothesis:[tex]\mu \leq 50[/tex]  

Alternative hypothesis:[tex]\mu > 50[/tex]  

The statistic for this case is given by:

[tex]t=\frac{\bar X-\mu_o}{\frac{s}{\sqrt{n}}}[/tex]  (1)  

For this case we have also the p value calculated

[tex]p_v =0.11[/tex]

If we use any significance level lower than 10% we FAIL to reject the null hypothesis (pvalue>significance) and we can conclude that the true mean is not higher than the mean for New York otherwise if we use a significance level higher than 10% the conclusion would be the opposite.

Step-by-step explanation:

Information given

[tex]\bar X=101.1[/tex] represent the sample mean

[tex]s=29.3[/tex] represent the sample standard deviation

[tex]n=50[/tex] sample size  

[tex]\mu_o =50[/tex] represent the value that we want to test

t would represent the statistic

[tex]p_v[/tex] represent the p value

System of hypothesis

We want to test if the scouts in her troop sold more cookies than the average in New York City (50), the system of hypothesis would be:  

Null hypothesis:[tex]\mu \leq 50[/tex]  

Alternative hypothesis:[tex]\mu > 50[/tex]  

The statistic for this case is given by:

[tex]t=\frac{\bar X-\mu_o}{\frac{s}{\sqrt{n}}}[/tex]  (1)  

For this case we have also the p value calculated

[tex]p_v =0.11[/tex]

Conclusion

If we use any significance level lower than 10% we FAIL to reject the null hypothesis (pvalue>significance) and we can conclude that the true mean is not higher than the mean for New York otherwise if we use a significance level higher than 10% the conclusion would be the opposite.

A recreational equipment supplier inds that among orders that include tents, 40% also include sleeping mats. Only 5% of orders that do not include tents do include sleeping mats. Also, 20% of orders include tents. Determine the following probabilities:

a. The order includes sleeping mats.
b. The order includes a tent given it includes sleeping mats.

Answers

Answer:

  a.  12%

  b.  2/3

Step-by-step explanation:

Given data regarding orders that do and don't include tents and/or mats, you want to find the probabilities of (a) the order includes mats; (b) the order includes a tent given that it includes mats.

Two-way table

The given data is summarized in the attached two-way table. The value in blue is given in the problem statement. The values in purple are computed from values given in the problem:

  40% of the 20% of orders include a tent and mat = 8% of orders

  5% of the 80% of orders with no tent = 4% of orders include a mat

The numbers in black make the totals come out right.

a. Includes mat

The table shows that 8% +4% = 12% of all orders include sleeping mats.

  P(m) = 12%

b. Includes tent, given includes mats

8% of the 12% of orders that include mats also include a tent. The probability of an order including a tent given that it includes a mat is ...

  P(t|m) = 8%/12% = 2/3

Brainliest to whoever gets this correct , Describe the end behavior of the following function:

Answers

Answer:

graph starts high and ends high

Now, when you click ROLL, the simulator will keep track of the average of all rolls so far. For example, if you roll 2 and then 4, the average will be 3. ROLL sides What is the average after 10 rolls

Answers

Step-by-step explanation:

Suppose the results obtained are numbers from 1 to 10, then after 10 rolls, the average is

(1+2+3+4+5+6+7+8+9+10)/10

= 55/10

= 5.5

Answer:

5 , 3 , 3.5     C) The average would approach a set value.

Step-by-step explanation:

The first three numbers don't matter it could be anyone 1-6.

The last answer is C

Which symbol should go into the box to make the statement true?
|17| ? 17.
A.< B.> C.equal sign with a line through it D. =

Answers

Answer:

the 3rd option is the right one

I would assume they’re equal but the one with brackets I believe is multiply?

rewrite the following without an exponent (-7)-1​

Answers

Answer:

Step-by-step explanation:

-7-1=-8

Pollution begins to enter a lake at time t =0 at a rate​ (in gallons per​ hour) given by the formula f(t)​, where t is the time​ (in hours). At the same​ time, a pollution filter begins to remove the pollution at a rate g (t) as long as the pollution remains in the lake.

Required:
a. How much pollution is in the lake after 12 hours?
b.Use a graphing calculator to find the time when the rate that pollution enters the lake equals the rate the pollution is removed.
c. Find the amount of pollution in the lake at the time found in part b.
d.Use a graphing calculator to find the time when all the pollution has been removed from the lake.

Answers

Answer:

a) 71.25 gallons

b) 25 hrs

c) 105 gallons

d) 47.91 hrs

Step-by-step explanation:

Solution:-

- The rate at which the pollution enter the lake is expressed by a function f ( t ) as follows:

                            [tex]f ( t ) = 10*( 1 - e^-^0^.^5^t )[/tex]

- Where, the rate at which pollution is removed from the lake via a filter is expressed by a function g ( t ):

                             [tex]g ( t ) = 0.4t[/tex]

- We will denote ( x ) as the amount of population in lake at time ( t ).

- We will set up a linear first order ODE using rate of change of pollution ( x ) in the lake at any instance ( t ):

                            [tex]\frac{dx}{dt} = ( flow _ i_n ) - ( flow _ o_u_t )[/tex]

- The respective inflow and outflow of pollution from the lake are expressed by the functions f ( t ) and g ( t ). Hence, the ODE becomes:

                           [tex]\frac{dx}{dt} = f ( t ) - g ( t )\\\\\frac{dx}{dt} = 10*( 1 - e^-^0^.^5^t ) - 0.4t\\\\[/tex]

- Separate the variables and integrate from t = 0 to t = 12 hours.

                         [tex]x = \int\limits^1^2_0 [{10*( 1 - e^-^0^.^5^t ) - 0.4t} ] \, dt \\\\x ( 12 ) = [ 10 * ( t + 2e^-^0^.^5^t ) - 0.2t^2 ] \limits^1^2_0\\\\x ( 12 ) = [ 10 * ( 12 + 2e^-^6 ) - 0.2(12)^2 ] - [ 10* ( 0 + 1 ) - 0.2(0)^2 ]\\\\x ( 12 ) = 71.25 galls[/tex]

- We will use the graphing calculator to evaluate the time ( t ) at which the rate of inflow of pollution is equal to the rate at which pollution is removed from the lake. In other words solve the following equation:

                          [tex]f ( t ) = g ( t )\\\\10*( 1 - e^-^0^.^5^t ) = 0.4t[/tex]

- Solving for the above equation the following intersection point was observed:

                          t = 25 hours

- We will perform integration of the previously expressed ODE from t = 0 to t = 25 hrs.

                           [tex]x ( 25 ) = [ 10*(t - 2e^0^.^5^t ) - 0.2t^2 ]\limits^2^5 _ 0\\\\x ( 25 ) = [ 10*(25 - 2e^1^2^.^5 ) - 0.2(25)^2 ] - [ 20 ] \\\\x ( 25 ) = 105 galls[/tex]

- We will evaluated result of the integration from time t = 0 to time t = t set the amount of pollution ( x ) equal to zero:

                          [tex]x ( t ) = [ 10*( t - 2e^-^0^.^5^t ) - 0.2t^2 ]\limits^t_0\\\\x ( t ) = 10 + 20e^-^0^.^5^t - 0.2t^2 - 20[/tex]

- Use the derived result x ( t ) and feed it into the graphing calculator and solve for x ( t ) = 0; hence,

                       10 + 20e^( -0.5*t ) - 0.2t^2 - 20 = 0

- The answer is evaluated as t = 47.91 hr

                         

Consider a sampling distribution with p equals 0.11 and samples of size n each. Using the appropriate​ formulas, find the mean and the standard deviation of the sampling distribution of the sample proportion. a. For a random sample of size n equals 5000. b. For a random sample of size n equals 1000. c. For a random sample of size n equals 250.

Answers

Answer:

a) Mean 0.11 and standard deviation 0.0044.

b) Mean 0.11 and standard deviation 0.0099.

c) Mean 0.11 and standard deviation 0.0198

Step-by-step explanation:

Central Limit Theorem:

For a proportion p in a sample of size n, the sampling distribution of the sample proportion will be approximately normal with mean [tex]\mu = p[/tex] and standard deviation [tex]s = \sqrt{\frac{p(1-p)}{n}}[/tex]

In this question:

[tex]p = 0.11[/tex]

a. For a random sample of size n equals 5000.

Mean:

[tex]\mu = p = 0.11[/tex]

Standard deviation:

[tex]s = \sqrt{\frac{0.11*0.89}{5000}} = 0.0044[/tex]

Mean 0.11 and standard deviation 0.0044.

b. For a random sample of size n equals 1000.

Mean:

[tex]\mu = p = 0.11[/tex]

Standard deviation:

[tex]s = \sqrt{\frac{0.11*0.89}{1000}} = 0.0044[/tex]

Mean 0.11 and standard deviation 0.0099.

c. For a random sample of size n equals 250.

Mean:

[tex]\mu = p = 0.11[/tex]

Standard deviation:

[tex]s = \sqrt{\frac{0.11*0.89}{250}} = 0.0198[/tex]

Mean 0.11 and standard deviation 0.0198

When vector is added to vector , the resultant is vector such that . This resultant vector has components Rx=Ax+BxRx=Ax+Bx and Ry=Ay+ByRy=Ay+By. How do you determine the direction of , the angle the vector makes with the positive x-axis?

Answers

Answer:

a. When vector is added to vector , the resultant is vector such that its magnitude and direction is proportional to the resultant magnitude and direction of the component vectors.

b. ∅ = [tex]tan^{-1}[/tex](Rx/Ry)

Step-by-step explanation:

Vectors are used to express physical quantities with magnitude and direction. Vectors can be added by the parallelogram method or the triangular method of addition. the magnitude and direction of the resultant vector is usually proportional to the magnitude and direction of the component vectors.

The direction of the angle is found as

tan∅ = Rx/Ry

∅ = [tex]tan^{-1}[/tex](Rx/Ry)

Can someone plz help me solved this problem I need help plz help me! Will mark you as brainiest!

Answers

Answer:

- 8b³+5ab²+7a²b

Step-by-step explanation:

(32ab³ - 20a²b² - 28a³b)/(-4a)= - 8b³+5ab²+7a²b

3. Barbara was asked to create a set of numbers that contained only integers. Which of the sets does

NOT contain only integers?

А

12

B

-3, -4,600,

9, 100, - 4, 12, -6, 5

5. 3, -6, -14, 3.5, 7

с

24

12

D 22, -12.0, 9, -14, 28, 4

Answers

Answer:

C (5.3, -6, -14, 3.5, 7)

Step-by-step explanation:

Integers are the sets of positive and negative whole numbers including zero.

In set notation, we denote the set of integers using the letter [tex]\mattbb{Z}[/tex].

From the given options:

In Option C (5. 3, -6, -14, 3.5, 7), we have the number 3.5 and 5.3 which are not whole numbers. Therefore 3.5 and 5.3 are not integers.

Option C is the set which does not contain only integers.

The domain of the function is all
The range of the function is all

Answers

The domain of a function is the set of all the x terms of the function and the range of a function is the set of all the y terms of a function.

For example, take a look below.

The domain is the set of all x terms in each ordered pair and the

range will be the set of all the y terms in each ordered pair.

At the airport, d airplanes flew away, and b were still there. How many more airplanes stayed at the airport than flew away?

Answers

Answer:

b-d

Step-by-step explanation:

At the airport, d airplanes flew away, and b were still there.

Number of airplanes which flew away =d Number of airplanes which stayed at the airport =b

We are required to determine how many more airplanes stayed at the airport than flew away.

To do this, we subtract the number of airplanes which flew away from the number which stayed.

Therefore:

Number of more airplanes which stayed than flew away

=Number of airplanes which stayed-Number of airplanes which flew away

= b-d

At 5:00 am, here's what we know about two airplanes: Airplane #1 has an elevation of 38520 ft and is descending at the rate of 750 ft/min. Airplane #2 has an elevation of 9400 ft and is climbing at the rate of 550 ft/min. (1) Let t represent the time in minutes since 5:00 am, and let E represent the elevation in feet. Write an equation for the elevation of each plane in terms of t . plane #1: E ( t ) = plane #2: E ( t ) =

Answers

Answer:

Plane 1

[tex]y_{1} = 38250\,ft - \left(750\,\frac{ft}{min}\right)\cdot t[/tex]

Plane 2

[tex]y_{2} = 9400\,ft + \left(550\,\frac{ft}{min}\right)\cdot t[/tex]

Step-by-step explanation:

As each plane is travelling at constant speed, each equation can be modelled after this expression:

[tex]y = y_{o} + \dot y \cdot t[/tex]

Where:

[tex]y_{o}[/tex] - Intial elevation of the airplane, measured in feet.

[tex]\dot y[/tex] - Climbing/Descending rate of the airplane, measured in feet per minute. (Positive - Climbing, Negative - Descending)

[tex]t[/tex] - Time, measured in minutes.

Equations are described below:

Plane 1

[tex]y_{1} = 38250\,ft - \left(750\,\frac{ft}{min}\right)\cdot t[/tex]

Plane 2

[tex]y_{2} = 9400\,ft + \left(550\,\frac{ft}{min}\right)\cdot t[/tex]

The number of miles a motorcycle, X, will travel on one gallon of gasoline is modeled by a normal distribution with mean 44 and standard deviation 5. If Mike starts a journey with one gallon of gasoline in the motorcycle, find the probability that, without refueling, he can travel more than 50 miles. Round your answer to four decimal places.

Answers

Answer:

[tex]P(X>50)=P(\frac{X-\mu}{\sigma}>\frac{50-\mu}{\sigma})=P(Z>\frac{50-44}{5})=P(z>1.2)[/tex]

And we can find this probability using the normal standar distribution and with the complement rule we got:

[tex]P(z>1.2)=1-P(z<1.2) =1-0.8849= 0.1151[/tex]

Step-by-step explanation:

Let X the random variable that represent the number of miles a motorcycle of a population, and for this case we know the distribution for X is given by:

[tex]X \sim N(44,5)[/tex]  

Where [tex]\mu=44[/tex] and [tex]\sigma=5[/tex]

We are interested on this probability

[tex]P(X>50)[/tex]

And we can use the z score formula given by:

[tex]z=\frac{x-\mu}{\sigma}[/tex]

And using this formula we got:

[tex]P(X>50)=P(\frac{X-\mu}{\sigma}>\frac{50-\mu}{\sigma})=P(Z>\frac{50-44}{5})=P(z>1.2)[/tex]

And we can find this probability using the normal standar distribution and with the complement rule we got:

[tex]P(z>1.2)=1-P(z<1.2) =1-0.8849= 0.1151[/tex]

Answer true or false to each statement concerning a confidence interval for a population mean.a) The length of a confidence interval can be determined if you know only the margin of error.b) The margin of error can be determined if you know only the length of the confidence interval.c) The confidence interval can be obtained if you know only the margin of error.d) The confidence interval can be obtained if you know only the margin of error and the sample mean.e) The margin of error can be determined if you know only the confidence level.f) The confidence level can be determined if you know only the margin of error.g) The margin of error can be determined if you know only the confidence level, population standard deviation, and sample size.h) The confidence level can be determined if you know only the margin of error, population standard deviation, and sample size.

Answers

Answer:

The true statements Include

A) The length of a confidence interval can be determined if you know only the margin of error.

B) The margin of error can be determined if you know only the length of the confidence interval.

D) The confidence interval can be obtained if you know only the margin of error and the sample mean.

G) The margin of error can be determined if you know only the confidence level, population standard deviation, and sample size.

F) The confidence level can be determined if you know only the margin of error, population standard deviation, and sample size.

The false statements include

C) The confidence interval can be obtained if you know only the margin of error.

E) The margin of error can be determined if you know only the confidence level.

F) The confidence level can be determined if you know only the margin of error.

Step-by-step explanation:

Confidence Interval for the population mean is basically an interval of range of values where the true population mean can be found with a certain level of confidence.

Mathematically,

Confidence Interval = (Sample mean) ± (Margin of error)

Margin of Error is the width of the confidence interval about the mean.

It is given mathematically as,

Margin of Error = (Critical value) × (standard Error of the mean)

The critical value is obtained using the confidence level (and sample size for t-distributions).

Standard Error of the mean = (Standard deviation)/[√(sample size)]

Standard error of the mean = σₓ = (σ/√n)

Taking each of the statements one at a time

a) The length of a confidence interval can be determined if you know only the margin of error.

According to the mathematical expression of the confidence interval, the length of the confidence interval is 2 × Margin of error. Hence, the length of the confidence interval can be truly obtained from only the margin of error.

This statement is true.

b) The margin of error can be determined if you know only the length of the confidence interval.

This is another way of phrasing statement A. The margin of error is half of the length of the confidence interval.

Hence, this statement is also true.

c) The confidence interval can be obtained if you know only the margin of error.

The confidence interval can be obtained from an expression that involves the sample mean and margin of error.

Confidence Interval = (Sample mean) ± (Margin of error)

The confidence interval cannot be obtained from just the margin of error. Hence, this statement is false.

d) The confidence interval can be obtained if you know only the margin of error and the sample mean.

Like o stated in (c) above, the confidence interval can be obtained from an expression that involves the sample mean and margin of error.

Confidence Interval = (Sample mean) ± (Margin of error)

Hence, this statement is true.

e) The margin of error can be determined if you know only the confidence level.

To obtain the margin of error, if the confidence interval isn't used, an expression involving the critical value and the standard error of the mean is used.

Margin of Error = (Critical value) × (standard Error of the mean)

The critical value is obtained using the confidence level, but the margin of error cannot be obtained from just that, we still need the standard error of the mean.

Hence, this statement is false.

f) The confidence level can be determined if you know only the margin of error.

This is another way of phrasing statement E. Hence, this statement too is false.

g) The margin of error can be determined if you know only the confidence level, population standard deviation, and sample size.

This is true as these is all that is required to obtain the margin of error.

h) The confidence level can be determined if you know only the margin of error, population standard deviation, and sample size.

This can be a bit stressful to obtain, but it is true. It is easy to see this from all the explanation from the beginning to this point.

Hope this Helps!!!

If (x + 5) - 2(4x - 1) = 0, what is the value of x?

Answers

Answer:

x=1

Step-by-step explanation:

(x + 5) - 2(4x - 1) = 0

Distribute

x+5 - 8x +2 =0

Combine like terms

-7x +7 = 0

Add 7x to each side

-7x+7 +7x = 7x

7 = 7x

divide by 7

7/7 = 7x/7

1 =x

What is the surface area of the figure shown?

Answers

Answer:

90 square feet.

Step-by-step explanation:

Let's set up an equation!

SA= 2(xy)+2(xz)+2(yz).

Lets say that x=3 y=6 and z=3

We plug it in.

2(18)+2(9)+2(18)= SA

36+18+36=SA

72+18=SA

90=SA

90 square feet = Surface area.

Let V be a vector space, and let v1, v2, v3 and v4 be linearly independent vectors in V . In parts (a) and (b) below, determine whether each of the given sets of vectors is linearly independent or linearly dependent. Prove your answers using the definition.
u1 = v1 − 7v3, u2 = v1, u3 = v3, u4 = v3 + v4.

Answers

Answer:

Step-by-step explanation:

[tex]u_3 = v_3[/tex] and  [tex]u_2 = v_1[/tex]

[tex]u_1= v_1 -7 v_3[/tex]

Replace the first line to the second line, you have [tex]u_1= u_2-7u_3[/tex]

That shows [tex]\{u_1, u_2, u_3,u_4\}[/tex]

is not linear independent.

Juanita bought a $9000.00 savings bond that pays 4.75% annual simple interest. When she cashes in the bond, she will have to pay 16% federal income tax on the earned interest. How much money will she net if she cashes in the bond after 6 years?

Answers

Answer:

$11154.6

Step-by-step explanation:

We have that the interest would be calculated as follows:  

9000 * 0.0475 = 427.5

then this for the amount of time in years:

427.5 * 6 = 2565

Now, we have to pay 16% federal interest tax. So, that would be

2565 * 0.16 = 410.4

Therefore, the money available to us after 6 years would be:

9000 + 2565 - 410.4 = 11154.6

It means that it would be $ 11154.6

Find the value of b in the graph of y=3x+b if it is known that the graph goes through the point: M(2,1)

Answers

Answer:

-5

Step-by-step explanation:

y=3x+b

M(2,1)

1=3*2+bb=1-6b= -5

what is the area of the figure composed of a parallelogram, a square, and a rectangle QUUUIIIIKKKKK!!

Answers

Answer:

126 inches

Step-by-step explanation:

The rectangle is 75 in.

The square is 16 in.

The parallelogram is 35 in.

I hope this helps :-)

A DVD is on sale for $1.05 off. This is a 15% discount from the original price. Use an equation to find the origina
price
O $16.05
$15.75
$10.50
O $7.00

Answers

Answer:

D. $7.00

Step-by-step explanation:

I just did it.

State the null and alternative hypotheses to test if the population represented by Sample 1 has a mean that is 2.0 units higher than the population represented by Sample 2. Let µd be the population mean of​ matched-pair differences for Sample 1 - Sample 2. State the null and alternative hypotheses.

Pair 1 2 3 4 5 6 7
Sample1 5 6 10 3 6 7 8
Sample2 4 4 3 5 5 5 3

Answers

Answer:

Null Hypothesis, [tex]H_0[/tex] : [tex]\mu_A-\mu_B[/tex] = 2.0 units  or  [tex]\mu_D[/tex] = 2.0 units

Alternate Hypothesis, [tex]H_A[/tex] : [tex]\mu_A-\mu_B\neq[/tex] 2.0 units  or  [tex]\mu_D\neq[/tex] 2.0 units

Step-by-step explanation:

We are given the following data below and we have test if the population represented by Sample 1 has a mean that is 2.0 units higher than the population represented by Sample 2;

Pair : 1 2 3 4 5 6 7

Sample1 : 5 6 10 3 6 7 8

Sample2 : 4 4 3 5 5 5 3

Let [tex]\mu_A[/tex] = population mean represented by Sample 1

[tex]\mu_B[/tex] = population mean represented by Sample 2

[tex]\mu_D[/tex] = population mean matched-pair differences for Sample 1 - Sample 2

So, Null Hypothesis, [tex]H_0[/tex] : [tex]\mu_A-\mu_B[/tex] = 2.0 units  or  [tex]\mu_D[/tex] = 2.0 units

Alternate Hypothesis, [tex]H_A[/tex] : [tex]\mu_A-\mu_B\neq[/tex] 2.0 units  or  [tex]\mu_D\neq[/tex] 2.0 units

Here null hypothesis states that the population represented by Sample 1 has a mean that is 2.0 units higher than the population represented by Sample 2.

On the other hand, alternate hypothesis states that the population represented by Sample 1 has a mean that is not 2.0 units higher than the population represented by Sample 2.

A password for a website must have 4 different digits. What is the probability a password chosen at random is 7654?
Please show work thanks

Answers

The 1st digit is 1 out of 10 numbers

The 2nd digit is 1 out of 9 numbers

The 3rd digit is 1 out of 8 numbers

The 4th digit is 1 out of 1 numbers.

Total combinations are 10 x 9 x 8 x 7 = 5040

Since the combination 7654 is one out of the total the probability would be 1/5040

HELP ASAP!! what is the value of f(-3)

Answers

Answer:

f(-3) = 4

Step-by-step explanation:

There are two ways to do it with the information given to you, you can look at the graph or use the equation.

Equation:

Since x = -3, you would put -3 in for x in the first equation which gives 4. It looks like you were on the right track but forgot that it was negative so you ended up adding 3 instead of subtracting. At least that is where I assume the 10 comes from.

Graphical:

Looking at the graph you can also find the value. You want to find the value of y at x = -3, so you look at the spot where x = -3. You will see that there are two "points" there, one open and one closed. The closed circle one means that the graph includes -3 in its x value. The open circle one means the graph contains every number until x=-3.

So to find the y value at x = -3 you want to look at the y value that corresponds to the closed circle of the graph since it is the part that includes x=-3 and you will see that y = 4 at that spot.

Match the building blocks of geometry to the statement that defines it

Answers

Answer:

Step-by-step explanation:

Diagram : A visual tool representing mathematical ideas to be interpreted.

Postulate : A mathematical statement taken as a fact.

Theorem : A mathematical statement proven using postulates and definitions.

Definition : A formal statement declaring the meaning of a word.

Ravi is 1 1/4 (mixed fraction).dinesh is 1 1/15 times as tall as Ravi .what is Dinesh's height

Answers

Dinesh is 1 1/3

First: Convert any mixed numbers to fractions.
Then your initial equation becomes: 5/4 times 16/15

Then :Applying the fractions formula for multiplication
5*16 =80
4*15. =60


Last:Simplifying 80/60, the answer is
1 1/3

Diastolic blood pressures are assumed to follow a normal distribution with a mean of 85 and a standard deviation of 12. What proportion of people has diastolic blood pressures less than 80?

Answers

Answer:

0.3745

Step-by-step explanation:

We have to solve the problem by calculating the z-score value that has the following formula:

z <(x - m) / sd

x is the value to evaluate (<80), m is the mean (85) and the standard deviation is sd (12)

replacing:

p (x <80) = z <(80 - 85) / 12

z <-0.416, we look for this value in the normal distribution table and it corresponds to:

p (x <80) = 0.3745

Which means that the proportion of people is 0.3745

Other Questions
Helps please......... Which macaw has a mass closest to 1 kilogram?PLS HELP BEST ANSWER GETS BRAINLESST AND FOLLOW!! SIROM Scientific Solutions has $10 million of outstanding equity and $5 million of bank debt. The bank debt costs 5% per year. The estimated equity beta is 2. If the market risk premium is 9% and the risk-free rate is 3%, compute the weighted average cost of capital if the firm's tax rate is 30%. Please answer correctly !!!!!!!! Will mark brainliest !!!!!!!!!!!!!! write a quadratic function f whose zeros are -2 and -9 what is on the module 8 DBA for world history flvs Beta Corporation had net income of $325,000 and paid dividends to common stockholders of $39,000 in 2017. The weighted average number of shares outstanding in 2017 was 50,000 shares. Beta Corporation's common stock is selling for $52 per share on the New York Stock Exchange. Beta Corporation's price-earnings ratio is Naval writes out the chemical equation for photosynthesis. She includes two reacting molecules to the left of the arrow and two resulting molecules to the right.Light energy, above the arrowChlorophyll,to the left of the arrow Enzymes, to the left of the arrowATP, above the arrow The following purchase transactions occurred during August for Backcountry Kayak Excursions. Aug 1 Purchased Kevlar kayaks (equipment) for $471 on account from Gear Inc. Aug 6 Purchased kayak paddles (supplies) for $701 on account from Southland Co. Aug 14 Purchased life vests (supplies) for $3,588 on account from Gear Inc. Record these transactions in a purchases journal. If no entry is required, select "no entry" from the dropdown box. If an amount box does not require an entry, leave it blank.Purchases Journal Page 1Date Account Post Ref Accounts Supplies Other Accounts Post. Amount Credited Payable Dr. Dr. Ref Cr. Aug 1 6 14 A houseplant is placed on a window sill that gets direct, bright sunlightevery afternoon. The plant starts to grow toward the window What kindof response is described by situation?AA negative response to a stimulus6A learned behaviorcGravitropismDPhototropism April and Blake went running. Blakeran 4 miles farther than April. Addedtogether, they ran a total of 16 miles.How far did April run?X = April's distance4 + x = Blake's distance 2. How is it possible that atherosclerosis was linked to Anna's untimely death? Which of the following is NOT stored in the bone?Bone InsideA.calciumB.fatC.mineralsD.water PLEASE HELP!! Use triangle ROE to find the following. Leave the answer in fraction form (no decimal answer). Your textbook authors recommend explicitly defining any term that might be misunderstood by someone reading the research proposal. If the research proposal is being written chiefly for use and review by researchers, why is such a step necessary? Select the correct answer.Which person is vulnerable to identity theft?A.Beverley opens a line of credit to purchase a household appliance.B.Deborah fills out her income tax form and includes her Social Security number.C. Josiah misses three monthly car loan payments in a row.D.Randell uses a computer at a public library to view his bank account online.ResetNext Tyler has asked for a fish tank for his birthday. The tank he wants is a rectangular prism 20 inches long, 10.5 inches wide, and 12.5 inches tall. What is the volume of the fish tank? Balance the equation: FeSO4(s) + O2(g) + H2O(l) = Fe(OH)3 + H2SO4 what is the simplified form of x + 8 / 4 - x + 5 / 4A. x + 3 / 4B. x - 3 / 4C. 3 / 4D. 2x + 13 / 4 What is investigative journalism?A. A special type of journalism involved with uncovering the truthB. A special type of journalism focused on celebrity newsC. A special type of journalism that only produces documentariesD. A special type of journalism which features face-to-face interviews